0 Daumen
520 Aufrufe

ich bin mir unsicher, wie ich richtig zeigen soll, dass diese rekursiv defineirte Folge konvergiert:

$$x_{n+1}=\frac{1}{6}x_{n}^{2}+\frac{2}{3}$$ $$x_{0}=2$$

Würdet ihr diesen Ansatz "absegnen":

\( x_{n+1}-x_{n}=\frac{1}{6}\left(x_{n}^{2}-x_{n-1}^{2}\right) \)

Aber nun bin ich mir unsicher wie ich das xn+1 und das xn zusammenfassen kann, um eine Aussage darüber zu treffen ob es < bzw. > 0 ist....

Falls ich mit der Monotonie fertig sein sollte, müsste ich doch nur noch den Grenzwert berechnen und hätte dann die Konvergenz gezeigt, oder?


Avatar von

2 Antworten

+2 Daumen
 
Beste Antwort

Deinen Ansatz segne ich ab. Forme ihn weiter um in 1/6(xn+xn+1)(xn-xn-1) und zeige außerdem xn+1<xn. Dann ist dein Term positiv.

Jede nach unten beschränkte, monoton fallende Folge hat einen Grenzwert g.

Avatar von 123 k 🚀

Hi, vielen vielen Dank für deine Antwort und tut mir leid für meine relativ späte Rückmeldung.

ich habe mir nochmal Gedanken über die Aufgabe gemacht, bzw. über rekursiv definierte Folgen im Generellen und wollte noch mal nachfragen, ob ich mit dieser Vorgehensweise nicht schon Konvergenz gezeigt und den Grenzwert in einem berechnet hätte:

$$x_{n+1}<x_{n}$$


⇔ $$\frac{1}{6}x^{2}_{n}+\frac{2}{3}-x_{n}<0$$



x1;2= 3±\( \sqrt{5} \) Innerhalb dieses Intervalls ist die Folge monoton fallend

Da 3-\( \sqrt{5} \)<2<3+\( \sqrt{5} \) fallt Sie monoton und strebt aufgrund der Begrenzung des Intervalls gegen den Grenzwert 3-\( \sqrt{5} \)

Kann man das so stehen lassen oder habe ich mit dieser Herangehensweise evt. etwas vergessen?
LG

@Roland

Forme ihn weiter um in 1/6(xn+xn+1)(xn-xn-1)

sollte wohl eher "...  in 1/6(x+ xn-1)(x- xn-1) " lauten.

und zeige außerdem xn+1<xn. Dann ist dein Term positiv.

welcher Term ist positiv

@Eichhörnchen

Da 3-√5 < 2 < 3+√5 fallt Sie monoton und strebt aufgrund der Begrenzung des Intervalls gegen den Grenzwert 3–√5

woher nimmst du die Überzeugung, dass alle Folgenglieder in diesem Intervall liegen?

Ich denke, du solltest dich an die nach deiner Meinung "schlechtere Antwort" von Spacko halten, weil Roland sich - im Gegensatz zu Spacko in (2)  - über die Vorgehensweise hierzu ausschweigt:

und zeige außerdem xn+1 < xn

Ich habe mir gedacht, dass sie doch nicht mehr aus diesem  Intervall herauskommen kann, weil sie sich ja immer dem Grenzwert 3-√5 nähert.

Aber wenn ich so drüber nachdenke, klingt das sehr plausibel, dass ich das erst noch beweisen muss. Ich müssze jetzt induktiv vorgehen, oder die Rekursionsformel anwenden, oder?

weil sie sich ja immer dem Grenzwert 3-√5 nähert.

Du müsstest doch erst einmal zeigen, dass nur der Grenzwert 3-√5 überhaupt in Frage kommt (Spacko (4)), obwohl der GW eigentlich nicht gefragt ist. Deshalb könntest du nach Spacko (3) sogar aufhören.

Ich müsste jetzt induktiv vorgehen ... 

Ja, so sehe ich das auch  (Spacko (2))

+2 Daumen

(1)  Man zeigt leicht, dass \(x_n>0\) für alle \(n\) ist.

(2)  Zeige per Induktion über \(n\), dass die Folge streng monoton fällt:
Dass \(x_1<x_0\) ist, rechnet man leicht nach. Wenn nun \(x_n<x_{n-1}\) für ein \(n\ge1\) gilt, dann gilt wg. (1) auch$$x_{n+1}-x_n=\tfrac16(x_n^2-x_{n-1}^2)<0.$$(3)  Die Folge ist nach unten beschränkt und monoton fallend, also konvergent.

(4)  Berechne den Grenzwert \(g<x_0\) aus \(g=\tfrac16g^2+\tfrac23\).

Avatar von

Daumen von mir (leider nicht ausgleichende Gerechtigkeit), obwohl du ja keinerlei Wert auf irgendwelche Punkte usw. legst  :-)

Ein anderes Problem?

Stell deine Frage

Willkommen bei der Mathelounge! Stell deine Frage einfach und kostenlos

x
Made by a lovely community